Quadrilateral
N
V
D
I
NVDI can be mapped onto Quadrilateral
F
L
S
W
FLSW by a reflection. If
m

V
=
2
3

m∠V=23

and
m

D
=
8
1

m∠D=81

, find
m

S
m∠S.

Quadrilateral NVDINVDI Can Be Mapped Onto Quadrilateral FLSWFLSW By A Reflection. If MV=23mV=23 And MD=81mD=81

Answers

Answer 1

9514 1404 393

Answer:

  ∠S = 81°

Step-by-step explanation:

Reflection does not change the angle measures.

The quadrilateral names tell you angle D corresponds with angle S. So, angle S has the same measure.

  ∠S = ∠D = 81°


Related Questions

HELP HELP HELP
Solve this

Answers

Answer:

What is the cos theta for, i would use sin to solve for theta and then we would get 41.25 degrees.

Step-by-step explanation:

Complete the table of ordered pairs for the linear y=2x-8

Answers

Given:

Consider the below figure attached with this question.

The linear equation is:

[tex]y=2x-8[/tex]

To find:

The values to complete the table of ordered pairs for the given linear equation.

Solution:

We have,

[tex]y=2x-8[/tex]

Substituting [tex]x=0[/tex] in the given equation, we get

[tex]y=2(0)-8[/tex]

[tex]y=0-8[/tex]

[tex]y=-8[/tex]

So, the value for first blank is -8.

Substituting [tex]y=-2[/tex] in the given equation, we get

[tex]-2=2x-8[/tex]

[tex]-2+8=2x[/tex]

[tex]\dfrac{6}{2}=x[/tex]

[tex]3=x[/tex]

So, the value for second blank is 3.

Substituting [tex]x=2[/tex] in the given equation, we get

[tex]y=2(2)-8[/tex]

[tex]y=4-8[/tex]

[tex]y=-4[/tex]

So, the value for third blank is -4.

Therefore, the required complete table is:

     x           y

     0         -8

    3         -2

     2          -4

Which number's estimate written as a single digit times a power of 10 will have a negative exponent?

Answers

105 i hope this helps if not then i’m sorry

There are 17 books on a shelf. 8 of these books are new. the rest of them are used. (GIVING BRAINLEST TO BEST ANSWER) what is the ratio?​

Answers

A. 9 to 8
B. 17 to 9?

Heyy!! Can someone help me please!!

3 (5x + 2) - 2 (4x -4)

I don’t know what to doooo!!

Answers

Answer:

7x + 14

Step-by-step explanation:

the first thing to do is expand the parentheses/brackets.

3(5x + 2) -2(4x - 4) will be

3(5x) + 3(2) -2(4x) -2(-4)

= 15x + 6 - 8x + 8

collect like terms

15x - 8x + 6 + 8 = 7x + 14

the answer is 7x + 14

Answer:

3(5x+2)-2(4x-4)

15x+6-8x+8

15x-8x+6+8

7x+14

What is the range of f(x) = 3X?
O A. All real numbers greater than 3
O B. All real numbers greater than or equal to 3
O C. All positive real numbers
O D. All real numbers

Answers

D all real numbers (-∞, ∞)

Find (x,y) such that (8,2)(x,y) = (28,22)

Answers

Answer:

(x,y) = (3.5, 11)

Step-by-step explanation:

(8,2)(x,y) = (28,22)

Dot product, so:

[tex](8,2)(x,y) = (8x,2y) = (28,22)[/tex]

Then

[tex]8x = 28[/tex]

[tex]x = \frac{28}{8} = 3.5[/tex]

And

[tex]2y = 22[/tex]

[tex]y = \frac{22}{2} = 11[/tex]

So

(x,y) = (3.5, 11)

A group of 3 boys is sharing 4 small pans of rice krispie treats. What is the total amount of rice krispie treats each boy will get?

Answers

Answer:

each boy will get 1.33333333333 of a pan

Step-by-step explanation:

Answer:

Each boy in that group would get about 1 and 1 third of a pan of treats or 1.33 repeating

Step-by-step explanation:

To find this you would take the numbers of pans (4) and divide it by the number of boys (3).

4 / 3 = 1.33333

Find the area a triangle with a base of 3 ft and a height of 2/3 ft

Answers

Answer:

1 ft^2

Step-by-step explanation:

The area of a triangle is given by

A = 1/2 bh  where b is the base and h is the height

A = 1/2 ( 3) * ( 2/3)

A = 3/2* 2/3

A =1

Answer:

1 ft^2

Step-by-step explanation:

The formula for the area of a triangle is as follows, a=b*h*1/2

Now, let's plug in the values to match your question. A=3*2/3*1/2

Lastly, solve. 3*2/3=6/3=2. Then divide 2 by 1/2=1.

Add the unit to make it 1 ft^2

Hope this helps!

A geneticist conducts an experiment with peas, one sample of offspring consisted of 450 green peas and 157 yellow peas. Based on these results, estimate the probability of getting an offspring pea that is green.

Answers

Answer: 0.738

Step-by-step explanation:

11. Mendelian Genetics. When Mendel conducted his famous genetics experiments with peas, one sample of offspring consisted of 428 green peas and 152 yellow peas. Based on those results, estimate the probability of getting an offspring pea that is green. Is the result reasonably close to the value that was expected?

p0 = 428/(428 + 152) = 0.737931

If you round your answer of 0.737931 to three decimals you will

get 0.738.

If f(x) = x -2 and g(x) = 2x – 6, then g(4)/f(3) =​

Answers

Answer:

Step-by-step explanation:

(2×4-6)/(3-2)=2

Answer:

[tex]{ \tt{f(x) = x - 2}} \\ { \bf{f(3) = 3 - 2 = 1}} \\ \\ { \tt{g(x) = 2x - 6}} \\ { \bf{g(4) = 2(4) - 6 = 2}} \\ \\ { \boxed{ \tt{ \frac{g(4)}{f(3)} = \frac{2}{1} = 2}}}[/tex]

 Marsha has a bag that contains 4 green marbles, 8 yellow marbles , and 20 red marbles . If she chooses one marble from the bag, what is the probability that the marble is not red?

PLEASE HELP IF YOURE GOOD AT GEOMETRY!!

Answers

Answer:

C. 3/8

HOPE THIS HELPS :)

Answer:

c. 3/8

Step-by-step explanation:

first you need the denomerator by adding all marbles together which equals 32. now for the munerator you need the sum of the green and yellow marbles. this equals 12. so your fraction is 12/32. next we simplify. we can divide both numbers by 4. getting us a fraction of 3/8.

Two workers paint lines for angled parking spaces

Answers

Answer:

2nd option

Step-by-step explanation:

The lines are parallel because the angles are congruent since they are Alternate exterior angles.

Which table represents a linear function

Answers

Answer:

First table

Step-by-step explanation:

Let V be the set of all 3x3 matrices with Real number entries, with the usual definitions of scalar multiplication and vector addition. Consider whether V is a vector space over C. Mark all true statements (there may be more than one).

a. The scalar closure axiom is satisfied
b. The additive inverse axiom is not satisfied
c The additive inverse axiom is satisfied
d. The additive closure axiom is not satisfied
e. The scalar closure axiom is not satisfied
f. The additive closure axiom is satsified
g. V is not a vector space over C
h. V is a vector space over C
i. The zero axiom is satisfied
j. The zero axiom is not satisfied

Answers

Answer:

the Scalar Closure axiom is not satisfiedV is not a Vector Space of CThe Additive Closure axiom is satisfied.

Step-by-step explanation:

According to the Question,

Given That, Let V be the set of all 3x3 matrices with Real number entries, with the usual definitions of scalar multiplication and vector addition. Consider whether V is a vector space over C.For V is a vector space over C and V is Set of 3x3 Matrices with Real entries.

Then, For any u,w ∈ V ⇒ u+w ∈ V

And u∈V and z∈C ⇒ z u ∈ V

So, If we take any z= i ∈ C

and V be any 3x3 matrices with Real entrices.

then, z,v ∉ V  ∴z,v Has Complex entries

So, the Scalar Closure axiom is not satisfied

Hence, V is not a Vector Space of C

Any u,w ∈ W ⇒ u+w ∈ V

So, The Additive Closure axiom is satisfied.

N/A questions does not exist

Answers

Answer:

ok

Step-by-step explanation:

why did you put it

Please please help me please I really need help please just tap on picture and you will see the question

Answers

Answer:

No , it is not a right angle triangle

Step-by-step explanation:

according to the pythagoras theorem in right angled triangle sum of square of two sides is equal to the square of it's hypotenuse.

using pythagoras theorem

a^2 + b^2 = c^2

9^2 + 16^2 = 25^2

81 + 256 = 625

337 = 625

since sum of square of two smallest sides of a triangle is not equal to the square of it's hypotenuse it can be concluded that the given figure does not form right angle triangle.

Help me with this answer I don’t it

Answers

Answer:

f(-2) = g(-2) this is the answer

How much is 12,856 ounces in pounds ?

Answers

Answer : 803.5 pounds

There are 16 ounces in a pound, so just divide 12,856 by 16, giving you 803.5.

Joe drives for 3 hours and covers 201 miles. In miles per hour, how fast was he driving?​

Answers

Answer:

67 mph

Step-by-step explanation:

201/3 = 67

Find the area of the following shape:

Answers

Answer:

36cm^2

Step-by-step explanation:

total area: 6x(4+3)=42

total area excluding the space: 42-(2x3)=36

Answer:

36 cm squared

Step-by-step explanation:

To solve this problem, I first construct a line. (shown in yellow in the first photo)

I then find the area of the top rectangle. (6 cm * 4 cm = 24 cm squared.)

Next, I find the area of the lower rectangle. But...to do that I have to find the length of the line that I constructed. To do this, I do  6cm-2cm=4cm.

Then I can find the area of the lower rectangle. (4cm*3cm=12cm squared.)

add up the area of both of the rectangles and.........12+24=36 cm squared

11x+7y=17
solve for y

Answers

[tex]\implies {\blue {\boxed {\boxed {\purple {\sf {\: y = \frac{17 - 11x}{7} }}}}}}[/tex]

[tex]\large\mathfrak{{\pmb{\underline{\red{Step-by-step\:explanation}}{\red{:}}}}}[/tex]

[tex]\\11x + 7y = 17[/tex]

[tex] \\➺ \: 7y = 17 - 11x[/tex]

[tex]\\➺ \: y = \frac{17 - 11x}{7} [/tex]

[tex]\bold{ \green{ \star{ \orange{Mystique35}}}}⋆[/tex]

Câu 1 (2 điểm). Cho hệ các vector

U = {(1,2,2); (0,-1,-1);(2,3,3);(1,0,0).

a) tìm số chiều và một cơ sở W của không gian con sinh bởi hệ vector U

b) tìm tham số k để u=(2,3,k^2 +1) là một tổ hợp tuyến tính cảu W, và suy ra [u]w

giải hộ mình với

Answers

Answer:

Step-by-step explanation:

What is the area of this figure?

Answers

Answer:

90km² only if it is parallelogram

Step-by-step explanation:

base = 9km

height=10km

area of parallelogram = b x h

=9km x 10km

=90km²

Answer:

A = 90km2

Step-by-step explanation:

Area of a rhombus is:

1. A = s x h (if given side and height)

2. A = 1/2 a x b (if given lengths of diagonals)

3. A = s^2 sin A (if given side and length)

Therefore from your problem, height and side is given thus, you'll use number 1

A = s x h

A = 9km x 10km = 90km2

your teacher for the discussion-based assessment.
In circle D, LEDH LEDG.
1. Determine the length of JG using
circle D.
Show your work and write out your
justification.
E
57"
9
Be prepared to answer questions
about additional angles, arcs and
segments from circle D.
F
D
J
H
669
G
2.
In OK, mZHKG=x+ 10 and
MZIKI = 3x - 22. Find m F).
74K
F
F
3. Find MLADB
OG with FA and FE tangent at A and E.
4. Find m2ABD
А
F
5. Find mzAFE
82
B
G
6. Find mLACE
E
1489
H Н
VOD

Answers

Answer:

3

Step-by-step explanation:

Find a power series for the function, centered at c, and determine the interval of convergence. f(x) = 9 3x + 2 , c = 6

Answers

Answer:

[tex]\frac{9}{3x + 2} = 1 - \frac{1}{3}(x - \frac{7}{3}) + \frac{1}{9}(x - \frac{7}{3})^2 - \frac{1}{27}(x - \frac{7}{3})^3 ........[/tex]

The interval of convergence is:[tex](-\frac{2}{3},\frac{16}{3})[/tex]

Step-by-step explanation:

Given

[tex]f(x)= \frac{9}{3x+ 2}[/tex]

[tex]c = 6[/tex]

The geometric series centered at c is of the form:

[tex]\frac{a}{1 - (r - c)} = \sum\limits^{\infty}_{n=0}a(r - c)^n, |r - c| < 1.[/tex]

Where:

[tex]a \to[/tex] first term

[tex]r - c \to[/tex] common ratio

We have to write

[tex]f(x)= \frac{9}{3x+ 2}[/tex]

In the following form:

[tex]\frac{a}{1 - r}[/tex]

So, we have:

[tex]f(x)= \frac{9}{3x+ 2}[/tex]

Rewrite as:

[tex]f(x) = \frac{9}{3x - 18 + 18 +2}[/tex]

[tex]f(x) = \frac{9}{3x - 18 + 20}[/tex]

Factorize

[tex]f(x) = \frac{1}{\frac{1}{9}(3x + 2)}[/tex]

Open bracket

[tex]f(x) = \frac{1}{\frac{1}{3}x + \frac{2}{9}}[/tex]

Rewrite as:

[tex]f(x) = \frac{1}{1- 1 + \frac{1}{3}x + \frac{2}{9}}[/tex]

Collect like terms

[tex]f(x) = \frac{1}{1 + \frac{1}{3}x + \frac{2}{9}- 1}[/tex]

Take LCM

[tex]f(x) = \frac{1}{1 + \frac{1}{3}x + \frac{2-9}{9}}[/tex]

[tex]f(x) = \frac{1}{1 + \frac{1}{3}x - \frac{7}{9}}[/tex]

So, we have:

[tex]f(x) = \frac{1}{1 -(- \frac{1}{3}x + \frac{7}{9})}[/tex]

By comparison with: [tex]\frac{a}{1 - r}[/tex]

[tex]a = 1[/tex]

[tex]r = -\frac{1}{3}x + \frac{7}{9}[/tex]

[tex]r = -\frac{1}{3}(x - \frac{7}{3})[/tex]

At c = 6, we have:

[tex]r = -\frac{1}{3}(x - \frac{7}{3}+6-6)[/tex]

Take LCM

[tex]r = -\frac{1}{3}(x + \frac{-7+18}{3}+6-6)[/tex]

r = -\frac{1}{3}(x + \frac{11}{3}+6-6)

So, the power series becomes:

[tex]\frac{9}{3x + 2} = \sum\limits^{\infty}_{n=0}ar^n[/tex]

Substitute 1 for a

[tex]\frac{9}{3x + 2} = \sum\limits^{\infty}_{n=0}1*r^n[/tex]

[tex]\frac{9}{3x + 2} = \sum\limits^{\infty}_{n=0}r^n[/tex]

Substitute the expression for r

[tex]\frac{9}{3x + 2} = \sum\limits^{\infty}_{n=0}(-\frac{1}{3}(x - \frac{7}{3}))^n[/tex]

Expand

[tex]\frac{9}{3x + 2} = \sum\limits^{\infty}_{n=0}[(-\frac{1}{3})^n* (x - \frac{7}{3})^n][/tex]

Further expand:

[tex]\frac{9}{3x + 2} = 1 - \frac{1}{3}(x - \frac{7}{3}) + \frac{1}{9}(x - \frac{7}{3})^2 - \frac{1}{27}(x - \frac{7}{3})^3 ................[/tex]

The power series converges when:

[tex]\frac{1}{3}|x - \frac{7}{3}| < 1[/tex]

Multiply both sides by 3

[tex]|x - \frac{7}{3}| <3[/tex]

Expand the absolute inequality

[tex]-3 < x - \frac{7}{3} <3[/tex]

Solve for x

[tex]\frac{7}{3} -3 < x <3+\frac{7}{3}[/tex]

Take LCM

[tex]\frac{7-9}{3} < x <\frac{9+7}{3}[/tex]

[tex]-\frac{2}{3} < x <\frac{16}{3}[/tex]

The interval of convergence is:[tex](-\frac{2}{3},\frac{16}{3})[/tex]

Given points (-3;-6), G(3; -2) and H(6; 1); determine:
(a) The equation of line FG

Answers

Answer:

The equation of line FG is [tex]y = \frac{2}{3}x - 4[/tex]

Step-by-step explanation:

Equation of a line:

The equation of a line has the following format:

[tex]y = mx + b[/tex]

In which m is the slope and b is the y-intercept.

F(-3;-6), G(3; -2)

When we have two points, the slope is given by the change in y divided by the change in x. So

Change in y : -2 - (-6) = -2 + 6 = 4

Change in x: 3 - (-3) = 3 + 3 = 6

Slope: [tex]m = \frac{4}{6} = \frac{2}{3}[/tex]

So

[tex]y = \frac{2}{3}x + b[/tex]

Finding b:

(3; -2) means that when [tex]x = 3, y = -2[/tex]. We use this to find b.

[tex]y = \frac{2}{3}x + b[/tex]

[tex]-2 = \frac{2}{3}(3) + b[/tex]

[tex]2 + b = -2[/tex]

[tex]b = -4[/tex]

The equation of line FG is [tex]y = \frac{2}{3}x - 4[/tex]

Which diagram represents the hypothesis of the converse of corresponding angles theorem?

Answers

Answer:

the first diagram

Step-by-step explanation:

first one

Which rules of exponents will be used to evallate this expression? Select three options.

Answers

I need the answer choices and the expression you’re referring to

Simplify:......................................................

Answers

Answer:

...

Step-by-step explanation:...

The cut off number is 2x-1
Other Questions
Dont need to send the math or anything just need the answer will give brainliest!!! A parabola can be represented by the equation x2 = 20y.What are the coordinates of the focus of the parabola? (5,0)(5,0)(0,5)(0,5) Which sentence accurately uses the homophone affect?A: I didnt know that my decision would affect you this much.B: The closing of the shop will have an affect on the whole town.C: I wonder if stricter rules will have an affect on student conduct.D: The new changes will immediately go into affect. Holtzman Clothiers's stock currently sells for $38 a share. It just paid a dividend of $1.5 a share (i.e., D0 = $1.5). The dividend is expected to grow at a constant rate of 4% a year.Required:a. What stock price is expected 1 year from now? b. What is the required rate of return? How to develop idea from Psalm 27:1 A principle that states in a sequence of undisturbed rocks, the oldest rocks are on the bottomand the rocks become progressively younger toward the top Choose from these answer:(Superposition)(Uniformity) (Fault) (Overfolding) I need help with my Spanish assignment asap Select a figure from the options which will replace the(?) as established by the problem figure HELP PLSS I CANT FAIL!!! Elements from Period 3 of the periodic table are highlighted. Which elementis a metalloid?A. SodiumB. ArgonC. SulfurD. Silicon Exercise 1: Look at the transcription and write the words.1. /dnjuri/ ..2. /februri/ ..3. /mrt/ ..4. /eprl / ..5. /me/ ..6. /du:n/ ..7. /dla/ ..8. /:gst/ ..9. /septembr/ ..10. /ktb(r)/ ..11. /novembr/ ..12. /dsembr The container of orange juice held 53 cups of juice. If a serving of orange juice is of a cup of orange juice, to thenearest whole serving, how many servings were in the container?05 servingsO 6 servings15 servings19 servingsMark this and returnSave and ExitNextSubmit The probability that Andrew has heart disease The two events are independent, so you need to findthe product of their probabilities: 0.9 x 0.75 = 0.675. Enter thecorrect answer The probability that Andrew has heart disease When should you include an analogy in a reflective essay? o when you want to add humor or excitement to the essay o when you want to talk about a reflection and an anecdote o when you want to show a well-known concept differently o when you want to clarify a confusing or difficult concept In the figure, ALM BLM by Side-Angle-Side (SAS). Which angles are congruent by CPCTC? Which foreign policy approach describes a country that avoids interaction with other countries? Intervention Diplomacy Imperialism Isolation if (x+y , 2x-3) = (5,3) find the value of x and y. plz answer ASAP!!! What is the correct form of the verb in the preterite tense.El profesor _______________ en la clase (entrar) Please help Ill give brainliest Consider the following simple regression model: y = 0 + 1 x 1 + u. In order to obtain consistent estimators of 0 and 1, when x and u are correlated, a new variable z is introduced into the model which satisfies the following two conditions: Cov( z, x) 0 and Cov ( z, u) = 0. The variable z is called a(n) _____ variable. dummy instrumental lagged dependent random can anyone tell me where is the perspective point ?